aboutsummaryrefslogtreecommitdiff
path: root/posts/2019-03-13-a-tail-of-two-densities.org
blob: a45c4a6ef206c9ab1c9ecb94aa1e57951bc5ebf2 (plain) (blame)
1
2
3
4
5
6
7
8
9
10
11
12
13
14
15
16
17
18
19
20
21
22
23
24
25
26
27
28
29
30
31
32
33
34
35
36
37
38
39
40
41
42
43
44
45
46
47
48
49
50
51
52
53
54
55
56
57
58
59
60
61
62
63
64
65
66
67
68
69
70
71
72
73
74
75
76
77
78
79
80
81
82
83
84
85
86
87
88
89
90
91
92
93
94
95
96
97
98
99
100
101
102
103
104
105
106
107
108
109
110
111
112
113
114
115
116
117
118
119
120
121
122
123
124
125
126
127
128
129
130
131
132
133
134
135
136
137
138
139
140
141
142
143
144
145
146
147
148
149
150
151
152
153
154
155
156
157
158
159
160
161
162
163
164
165
166
167
168
169
170
171
172
173
174
175
176
177
178
179
180
181
182
183
184
185
186
187
188
189
190
191
192
193
194
195
196
197
198
199
200
201
202
203
204
205
206
207
208
209
210
211
212
213
214
215
216
217
218
219
220
221
222
223
224
225
226
227
228
229
230
231
232
233
234
235
236
237
238
239
240
241
242
243
244
245
246
247
248
249
250
251
252
253
254
255
256
257
258
259
260
261
262
263
264
265
266
267
268
269
270
271
272
273
274
275
276
277
278
279
280
281
282
283
284
285
286
287
288
289
290
291
292
293
294
295
296
297
298
299
300
301
302
303
304
305
306
307
308
309
310
311
312
313
314
315
316
317
318
319
320
321
322
323
324
325
326
327
328
329
330
331
332
333
334
335
336
337
338
339
340
341
342
343
344
345
346
347
348
349
350
351
352
353
354
355
356
357
358
359
360
361
362
363
364
365
366
367
368
369
370
371
372
373
374
375
376
377
378
379
380
381
382
383
384
385
386
387
388
389
390
391
392
393
394
395
396
397
398
399
400
401
402
403
404
405
406
407
408
409
410
411
412
413
414
415
416
417
418
419
420
421
422
423
424
425
426
427
428
429
430
431
432
433
434
435
436
437
438
439
440
441
442
443
444
445
446
447
448
449
450
451
452
453
454
455
456
457
458
459
460
461
462
463
464
465
466
467
468
469
470
471
472
473
474
475
476
477
478
479
480
481
482
483
484
485
486
487
488
489
490
491
492
493
494
495
496
497
498
499
500
501
502
503
504
505
506
507
508
509
510
511
512
513
514
515
516
517
518
519
520
521
522
523
524
525
526
527
528
529
530
531
532
533
534
535
536
537
538
539
540
541
542
543
544
545
546
547
548
549
550
551
552
553
554
555
556
557
558
559
560
561
562
563
564
565
566
567
568
569
570
571
572
573
574
575
576
577
578
579
580
581
582
583
584
585
586
587
588
589
590
591
592
593
594
595
596
597
598
599
600
601
602
603
604
605
606
607
608
609
610
611
612
613
614
615
616
617
618
619
620
621
622
623
624
625
626
627
628
629
630
631
632
633
634
635
636
637
638
639
640
641
642
643
644
645
646
647
648
649
650
651
652
653
654
655
656
657
658
659
660
661
662
663
664
665
666
667
668
669
670
671
672
673
674
675
676
677
678
679
680
681
682
683
684
685
686
687
688
689
690
691
692
693
694
695
696
697
698
699
700
701
702
703
704
705
706
707
708
709
710
711
712
713
714
715
716
717
718
719
720
721
722
723
724
725
726
727
728
729
730
731
732
733
734
735
736
737
738
739
740
741
742
743
744
745
746
747
748
749
750
751
752
753
754
755
756
757
758
759
760
761
762
763
764
765
766
767
768
769
770
771
772
773
774
775
776
777
778
779
780
781
782
783
784
785
786
787
788
789
790
791
792
793
794
795
796
797
798
799
800
801
802
803
804
805
806
807
808
809
810
811
812
813
814
815
816
817
818
819
820
821
822
823
824
825
826
827
828
829
830
831
832
833
834
835
836
837
838
839
840
841
842
843
844
845
846
847
848
849
850
851
852
853
854
855
856
857
858
859
860
861
862
863
864
865
866
867
868
869
870
871
872
873
874
875
876
877
878
879
880
881
882
883
884
885
886
887
888
889
890
891
892
893
894
895
896
897
898
899
900
901
902
903
904
905
906
907
908
909
910
911
912
913
914
915
916
917
918
919
920
921
922
923
924
925
926
927
928
929
930
931
932
933
934
935
936
937
938
939
940
941
942
943
944
945
946
947
948
949
950
951
952
953
954
955
956
957
958
959
960
961
962
963
964
965
966
967
968
969
970
971
972
973
974
975
976
977
978
979
980
981
982
983
984
985
986
987
988
989
990
991
992
993
994
995
996
997
998
999
1000
1001
1002
1003
1004
1005
1006
1007
1008
1009
1010
1011
1012
1013
1014
1015
1016
1017
1018
1019
1020
1021
1022
1023
1024
1025
1026
1027
1028
1029
1030
1031
1032
1033
1034
1035
1036
1037
1038
1039
1040
1041
1042
1043
1044
1045
1046
1047
1048
1049
1050
1051
1052
1053
1054
1055
1056
1057
1058
1059
1060
1061
1062
1063
1064
1065
1066
1067
1068
1069
1070
1071
1072
1073
1074
1075
1076
1077
1078
1079
1080
1081
1082
1083
1084
1085
1086
1087
1088
1089
1090
1091
1092
1093
1094
1095
1096
1097
1098
1099
1100
1101
1102
1103
1104
1105
1106
1107
1108
1109
1110
1111
1112
1113
1114
1115
1116
1117
1118
1119
1120
1121
1122
1123
1124
1125
1126
1127
1128
1129
1130
1131
1132
1133
1134
1135
1136
1137
1138
1139
1140
1141
1142
1143
1144
1145
1146
1147
1148
1149
1150
1151
1152
1153
1154
1155
1156
1157
1158
1159
1160
1161
1162
1163
1164
1165
1166
1167
1168
1169
1170
1171
1172
1173
1174
1175
1176
1177
1178
1179
1180
1181
1182
1183
1184
1185
1186
1187
1188
1189
1190
1191
1192
1193
1194
1195
1196
1197
1198
1199
1200
1201
1202
1203
1204
1205
1206
1207
1208
1209
1210
1211
1212
1213
1214
1215
1216
1217
1218
1219
1220
1221
1222
1223
1224
1225
1226
1227
1228
1229
1230
1231
1232
1233
1234
1235
1236
1237
1238
1239
1240
1241
1242
1243
1244
1245
1246
1247
1248
1249
1250
1251
1252
1253
1254
1255
1256
1257
1258
1259
1260
1261
1262
1263
1264
1265
1266
1267
1268
1269
1270
1271
1272
1273
1274
1275
1276
1277
1278
1279
1280
1281
1282
1283
1284
1285
1286
1287
1288
1289
1290
1291
1292
1293
1294
1295
1296
1297
1298
1299
1300
1301
1302
1303
1304
1305
1306
1307
1308
1309
1310
1311
1312
1313
1314
1315
1316
1317
1318
1319
1320
1321
1322
1323
# Copyright (C) 2013-2021 Yuchen Pei.

# Permission is granted to copy, distribute and/or modify this
# document under the terms of the GNU Free Documentation License,
# Version 1.3 or any later version published by the Free Software
# Foundation; with no Invariant Sections, no Front-Cover Texts, and
# no Back-Cover Texts. You should have received a copy of the GNU
# Free Documentation License. If not, see <https://www.gnu.org/licenses/>.

# This work is licensed under the Creative Commons Attribution-ShareAlike 4.0 International License. To view a copy of this license, visit http://creativecommons.org/licenses/by-sa/4.0/ or send a letter to Creative Commons, PO Box 1866, Mountain View, CA 94042, USA.

#+title: A Tail of Two Densities

#+date: <2019-03-13>

This is Part 1 of a two-part post where I give an introduction to the
mathematics of differential privacy.

Practically speaking,
[[https://en.wikipedia.org/wiki/Differential_privacy][differential
privacy]] is a technique of perturbing database queries so that query
results do not leak too much information while still being relatively
accurate.

This post however focuses on the mathematical aspects of differential
privacy, which is a study of
[[https://en.wikipedia.org/wiki/Concentration_inequality][tail bounds]]
of the divergence between two probability measures, with the end goal of
applying it to
[[https://en.wikipedia.org/wiki/Stochastic_gradient_descent][stochastic
gradient descent]]. This post should be suitable for anyone familiar
with probability theory.

I start with the definition of \(\epsilon\)-differential privacy
(corresponding to max divergence), followed by
\((\epsilon, \delta)\)-differential privacy (a.k.a. approximate
differential privacy, corresponding to the \(\delta\)-approximate max
divergence). I show a characterisation of the
\((\epsilon, \delta)\)-differential privacy as conditioned
\(\epsilon\)-differential privacy. Also, as examples, I illustrate the
\(\epsilon\)-dp with Laplace mechanism and, using some common tail bounds,
the approximate dp with the Gaussian mechanism.

Then I continue to show the effect of combinatorial and sequential
compositions of randomised queries (called mechanisms) on privacy by
stating and proving the composition theorems for differential privacy,
as well as the effect of mixing mechanisms, by presenting the
subsampling theorem (a.k.a. amplification theorem).

In [[/posts/2019-03-14-great-but-manageable-expectations.html][Part 2]],
I discuss the Rényi differential privacy, corresponding to the Rényi
divergence, a study of the
[[https://en.wikipedia.org/wiki/Moment-generating_function][moment
generating functions]] of the divergence between probability measures to
derive the tail bounds.

Like in Part 1, I prove a composition theorem and a subsampling theorem.

I also attempt to reproduce a seemingly better moment bound for the
Gaussian mechanism with subsampling, with one intermediate step which I
am not able to prove.

After that I explain the Tensorflow implementation of differential
privacy in its
[[https://github.com/tensorflow/privacy/tree/master/privacy][Privacy]]
module, which focuses on the differentially private stochastic gradient
descent algorithm (DP-SGD).

Finally I use the results from both Part 1 and Part 2 to obtain some
privacy guarantees for composed subsampling queries in general, and for
DP-SGD in particular. I also compare these privacy guarantees.

*Acknowledgement*. I would like to thank
[[http://stockholm.ai][Stockholm AI]] for introducing me to the subject
of differential privacy. Thanks to Amir Hossein Rahnama for hosting the
discussions at Stockholm AI. Thanks to (in chronological order) Reynaldo
Boulogne, Martin Abedi, Ilya Mironov, Kurt Johansson, Mark Bun, Salil
Vadhan, Jonathan Ullman, Yuanyuan Xu and Yiting Li for communication and
discussions. Also thanks to the
[[https://www.reddit.com/r/MachineLearning/][r/MachineLearning]]
community for comments and suggestions which result in improvement of
readability of this post. The research was done while working at
[[https://www.kth.se/en/sci/institutioner/math][KTH Department of
Mathematics]].

/If you are confused by any notations, ask me or try
[[/notations.html][this]]. This post (including both Part 1 and Part2)
is licensed under [[https://creativecommons.org/licenses/by-sa/4.0/][CC
BY-SA]] and [[https://www.gnu.org/licenses/fdl.html][GNU FDL]]./

** The gist of differential privacy
   :PROPERTIES:
   :CUSTOM_ID: the-gist-of-differential-privacy
   :ID:       91bf2eb5-8509-4180-b471-939280dc1438
   :END:
If you only have one minute, here is what differential privacy is about:

Let \(p\) and \(q\) be two probability densities, we define the /divergence
variable/[fn:1] of \((p, q)\) to be

\[L(p || q) := \log {p(\xi) \over q(\xi)}\]

where \(\xi\) is a random variable distributed according to \(p\).

Roughly speaking, differential privacy is the study of the tail bound of
\(L(p || q)\): for certain \(p\)s and \(q\)s, and for \(\epsilon > 0\), find
\(\delta(\epsilon)\) such that

\[\mathbb P(L(p || q) > \epsilon) < \delta(\epsilon),\]

where \(p\) and \(q\) are the laws of the outputs of a randomised functions
on two very similar inputs. Moreover, to make matters even simpler, only
three situations need to be considered:

1. (General case) \(q\) is in the form of \(q(y) = p(y + \Delta)\) for some
   bounded constant \(\Delta\).
2. (Compositions) \(p\) and \(q\) are combinatorial or sequential
   compositions of some simpler \(p_i\)'s and \(q_i\)'s respectively
3. (Subsampling) \(p\) and \(q\) are mixtures / averages of some simpler
   \(p_i\)'s and \(q_i\)'s respectively

In applications, the inputs are databases and the randomised functions
are queries with an added noise, and the tail bounds give privacy
guarantees. When it comes to gradient descent, the input is the training
dataset, and the query updates the parameters, and privacy is achieved
by adding noise to the gradients.

Now if you have an hour...

** \(\epsilon\)-dp
   :PROPERTIES:
   :CUSTOM_ID: epsilon-dp
   :ID:       d29da3db-8b9a-4bad-811e-4af1cd9f856d
   :END:
*Definition (Mechanisms)*. Let \(X\) be a space with a metric
\(d: X \times X \to \mathbb N\). A /mechanism/ \(M\) is a function that
takes \(x \in X\) as input and outputs a random variable on \(Y\).

In this post, \(X = Z^m\) is the space of datasets of \(m\) rows for some
integer \(m\), where each item resides in some space \(Z\). In this case the
distance \(d(x, x') := \#\{i: x_i \neq x'_i\}\) is the number of rows that
differ between \(x\) and \(x'\).

Normally we have a query \(f: X \to Y\), and construct the mechanism \(M\)
from \(f\) by adding a noise:

\[M(x) := f(x) + \text{noise}.\]

Later, we will also consider mechanisms constructed from composition or
mixture of other mechanisms.

In this post \(Y = \mathbb R^d\) for some \(d\).

*Definition (Sensitivity)*. Let \(f: X \to \mathbb R^d\) be a function.
The /sensitivity/ \(S_f\) of \(f\) is defined as

\[S_f := \sup_{x, x' \in X: d(x, x') = 1} \|f(x) - f(x')\|_2,\]

where \(\|y\|_2 = \sqrt{y_1^2 + ... + y_d^2}\) is the \(\ell^2\)-norm.

*Definition (Differential Privacy)*. A mechanism \(M\) is called
\(\epsilon\)/-differential privacy/ (\(\epsilon\)-dp) if it satisfies the
following condition: for all \(x, x' \in X\) with \(d(x, x') = 1\), and for
all measureable set \(S \subset \mathbb R^n\),

\[\mathbb P(M(x) \in S) \le e^\epsilon P(M(x') \in S). \qquad (1)\]

Practically speaking, this means given the results from perturbed query
on two known databases that differs by one row, it is hard to determine
which result is from which database.

An example of \(\epsilon\)-dp mechanism is the Laplace mechanism.

*Definition*. The /Laplace distribution/ over \(\mathbb R\) with parameter
\(b > 0\) has probability density function

\[f_{\text{Lap}(b)}(x) = {1 \over 2 b} e^{- {|x| \over b}}.\]

*Definition*. Let \(d = 1\). The /Laplace mechanism/ is defined by

\[M(x) = f(x) + \text{Lap}(b).\]

*Claim*. The Laplace mechanism with

\[b \ge \epsilon^{-1} S_f \qquad (1.5)\]

is \(\epsilon\)-dp.

*Proof*. Quite straightforward. Let \(p\) and \(q\) be the laws of \(M(x)\)
and \(M(x')\) respectively.

\[{p (y) \over q (y)} = {f_{\text{Lap}(b)} (y - f(x)) \over f_{\text{Lap}(b)} (y - f(x'))} = \exp(b^{-1} (|y - f(x')| - |y - f(x)|))\]

Using triangular inequality \(|A| - |B| \le |A - B|\) on the right hand
side, we have

\[{p (y) \over q (y)} \le \exp(b^{-1} (|f(x) - f(x')|)) \le \exp(\epsilon)\]

where in the last step we use the condition (1.5). \(\square\)

** Approximate differential privacy
   :PROPERTIES:
   :CUSTOM_ID: approximate-differential-privacy
   :ID:       c48c68f8-d749-47f8-b5de-c92cc53f8cea
   :END:
Unfortunately, \(\epsilon\)-dp does not apply to the most commonly used
noise, the Gaussian noise. To fix this, we need to relax the definition
a bit.

*Definition*. A mechanism \(M\) is said to be
\((\epsilon, \delta)\)/-differentially private/ if for all \(x, x' \in X\)
with \(d(x, x') = 1\) and for all measureable \(S \subset \mathbb R^d\)

\[\mathbb P(M(x) \in S) \le e^\epsilon P(M(x') \in S) + \delta. \qquad (2)\]

Immediately we see that the \((\epsilon, \delta)\)-dp is meaningful only
if \(\delta < 1\).

*** Indistinguishability
    :PROPERTIES:
    :CUSTOM_ID: indistinguishability
    :ID:       7875ad81-326b-4eaa-a3ae-9e09df96ea1b
    :END:
To understand \((\epsilon, \delta)\)-dp, it is helpful to study
\((\epsilon, \delta)\)-indistinguishability.

*Definition*. Two probability measures \(p\) and \(q\) on the same space are
called \((\epsilon, \delta)\)/-ind(istinguishable)/ if for all measureable
sets \(S\):

$$\begin{aligned}
p(S) \le e^\epsilon q(S) + \delta, \qquad (3) \\
q(S) \le e^\epsilon p(S) + \delta. \qquad (4)
\end{aligned}$$

As before, we also call random variables \(\xi\) and \(\eta\) to be
\((\epsilon, \delta)\)-ind if their laws are \((\epsilon, \delta)\)-ind.
When \(\delta = 0\), we call it \(\epsilon\)-ind.

Immediately we have

*Claim 0*. \(M\) is \((\epsilon, \delta)\)-dp (resp. \(\epsilon\)-dp) iff
\(M(x)\) and \(M(x')\) are \((\epsilon, \delta)\)-ind (resp. \(\epsilon\)-ind)
for all \(x\) and \(x'\) with distance \(1\).

*Definition (Divergence Variable)*. Let \(p\) and \(q\) be two probability
measures. Let \(\xi\) be a random variable distributed according to \(p\),
we define a random variable \(L(p || q)\) by

\[L(p || q) := \log {p(\xi) \over q(\xi)},\]

and call it the /divergence variable/ of \((p, q)\).

One interesting and readily verifiable fact is

\[\mathbb E L(p || q) = D(p || q)\]

where \(D\) is the
[[https://en.wikipedia.org/wiki/Kullback%E2%80%93Leibler_divergence][KL-divergence]].

*Claim 1*. If

$$\begin{aligned}
\mathbb P(L(p || q) \le \epsilon) &\ge 1 - \delta, \qquad(5) \\
\mathbb P(L(q || p) \le \epsilon) &\ge 1 - \delta
\end{aligned}$$

then \(p\) and \(q\) are \((\epsilon, \delta)\)-ind.

*Proof*. We verify (3), and (4) can be shown in the same way. Let
\(A := \{y \in Y: \log {p(y) \over q(y)} > \epsilon\}\), then by (5) we
have

\[p(A) < \delta.\]

So

\[p(S) = p(S \cap A) + p(S \setminus A) \le \delta + e^\epsilon q(S \setminus A) \le \delta + e^\epsilon q(S).\]

\(\square\)

This Claim translates differential privacy to the tail bound of
divergence variables, and for the rest of this post all dp results are
obtained by estimating this tail bound.

In the following we discuss the converse of Claim 1. The discussions are
rather technical, and readers can skip to the
[[#back-to-approximate-differential-privacy][next subsection]] on first
reading.

The converse of Claim 1 is not true.

*Claim 2*. There exists \(\epsilon, \delta > 0\), and \(p\) and \(q\) that are
\((\epsilon, \delta)\)-ind, such that

$$\begin{aligned}
\mathbb P(L(p || q) \le \epsilon) &< 1 - \delta, \\
\mathbb P(L(q || p) \le \epsilon) &< 1 - \delta
\end{aligned}$$

*Proof*. Here's a example. Let \(Y = \{0, 1\}\), and \(p(0) = q(1) = 2 / 5\)
and \(p(1) = q(0) = 3 / 5\). Then it is not hard to verify that \(p\) and
\(q\) are \((\log {4 \over 3}, {1 \over 3})\)-ind: just check (3) for all
four possible \(S \subset Y\) and (4) holds by symmetry. On the other
hand,

\[\mathbb P(L(p || q) \le \log {4 \over 3}) = \mathbb P(L(q || p) \le \log {4 \over 3}) = {2 \over 5} < {2 \over 3}.\]

\(\square\)

A weaker version of the converse of Claim 1 is true
(Kasiviswanathan-Smith 2015), though:

*Claim 3*. Let \(\alpha > 1\). If \(p\) and \(q\) are
\((\epsilon, \delta)\)-ind, then

\[\mathbb P(L(p || q) > \alpha \epsilon) < {1 \over 1 - \exp((1 - \alpha) \epsilon)} \delta.\]

*Proof*. Define

\[S = \{y: p(y) > e^{\alpha \epsilon} q(y)\}.\]

Then we have

\[e^{\alpha \epsilon} q(S) < p(S) \le e^\epsilon q(S) + \delta,\]

where the first inequality is due to the definition of \(S\), and the
second due to the \((\epsilon, \delta)\)-ind. Therefore

\[q(S) \le {\delta \over e^{\alpha \epsilon} - e^\epsilon}.\]

Using the \((\epsilon, \delta)\)-ind again we have

\[p(S) \le e^\epsilon q(S) + \delta = {1 \over 1 - e^{(1 - \alpha) \epsilon}} \delta.\]

\(\square\)

This can be quite bad if \(\epsilon\) is small.

To prove the composition theorems in the next section, we need a
condition better than that in Claim 1 so that we can go back and forth
between indistinguishability and such condition. In other words, we need
a /characterisation/ of indistinguishability.

Let us take a careful look at the condition in Claim 1 and call it *C1*:

*C1*. \(\mathbb P(L(p || q) \le \epsilon) \ge 1 - \delta\) and
\(\mathbb P(L(q || p) \le \epsilon) \ge 1 - \delta\)

It is equivalent to

*C2*. there exist events \(A, B \subset Y\) with probabilities \(p(A)\) and
\(q(B)\) at least \(1 - \delta\) such that
\(\log p(y) - \log q(y) \le \epsilon\) for all \(y \in A\) and
\(\log q(y) - \log p(y) \le \epsilon\) for all \(y \in B\).

A similar-looking condition to *C2* is the following:

*C3*. Let \(\Omega\) be the
[[https://en.wikipedia.org/wiki/Probability_space#Definition][underlying
probability space]]. There exist two events \(E, F \subset \Omega\) with
\(\mathbb P(E), \mathbb P(F) \ge 1 - \delta\), such that
\(|\log p_{|E}(y) - \log q_{|F}(y)| \le \epsilon\) for all \(y \in Y\).

Here \(p_{|E}\) (resp. \(q_{|F}\)) is \(p\) (resp. \(q\)) conditioned on event
\(E\) (resp. \(F\)).

*Remark*. Note that the events in *C2* and *C3* are in different spaces,
and therefore we can not write \(p_{|E}(S)\) as \(p(S | E)\) or \(q_{|F}(S)\)
as \(q(S | F)\). In fact, if we let \(E\) and \(F\) in *C3* be subsets of \(Y\)
with \(p(E), q(F) \ge 1 - \delta\) and assume \(p\) and \(q\) have the same
supports, then *C3* degenerates to a stronger condition than *C2*.
Indeed, in this case \(p_E(y) = p(y) 1_{y \in E}\) and
\(q_F(y) = q(y) 1_{y \in F}\), and so \(p_E(y) \le e^\epsilon q_F(y)\)
forces \(E \subset F\). We also obtain \(F \subset E\) in the same way. This
gives us \(E = F\), and *C3* becomes *C2* with \(A = B = E = F\).

As it turns out, *C3* is the condition we need.

*Claim 4*. Two probability measures \(p\) and \(q\) are
\((\epsilon, \delta)\)-ind if and only if *C3* holds.

*Proof*(Murtagh-Vadhan 2018). The "if" direction is proved in the same
way as Claim 1. Without loss of generality we may assume
\(\mathbb P(E) = \mathbb P(F) \ge 1 - \delta\). To see this, suppose \(F\)
has higher probability than \(E\), then we can substitute \(F\) with a
subset of \(F\) that has the same probability as \(E\) (with possible
enlargement of the probability space).

Let \(\xi \sim p\) and \(\eta \sim q\) be two independent random variables,
then

$$\begin{aligned}
p(S) &= \mathbb P(\xi \in S | E) \mathbb P(E) + \mathbb P(\xi \in S; E^c) \\
&\le e^\epsilon \mathbb P(\eta \in S | F) \mathbb P(E) + \delta \\
&= e^\epsilon \mathbb P(\eta \in S | F) \mathbb P(F) + \delta\\
&\le e^\epsilon q(S) + \delta.
\end{aligned}$$

The "only-if" direction is more involved.

We construct events \(E\) and \(F\) by constructing functions
\(e, f: Y \to [0, \infty)\) satisfying the following conditions:

1. \(0 \le e(y) \le p(y)\) and \(0 \le f(y) \le q(y)\) for all \(y \in Y\).
2. \(|\log e(y) - \log f(y)| \le \epsilon\) for all \(y \in Y\).
3. \(e(Y), f(Y) \ge 1 - \delta\).
4. \(e(Y) = f(Y)\).

Here for a set \(S \subset Y\), \(e(S) := \int_S e(y) dy\), and the same
goes for \(f(S)\).

Let \(\xi \sim p\) and \(\eta \sim q\). Then we define \(E\) and \(F\) by

$$\mathbb P(E | \xi = y) = e(y) / p(y) \\
\mathbb P(F | \eta = y) = f(y) / q(y).$$

*Remark inside proof*. This can seem a bit confusing. Intuitively, we
can think of it this way when \(Y\) is finite: Recall a random variable on
\(Y\) is a function from the probability space \(\Omega\) to \(Y\). Let event
\(G_y \subset \Omega\) be defined as \(G_y = \xi^{-1} (y)\). We cut \(G_y\)
into the disjoint union of \(E_y\) and \(G_y \setminus E_y\) such that
\(\mathbb P(E_y) = e(y)\). Then \(E = \bigcup_{y \in Y} E_y\). So \(e(y)\) can
be seen as the "density" of \(E\).

Indeed, given \(E\) and \(F\) defined this way, we have

\[p_E(y) = {e(y) \over e(Y)} \le {\exp(\epsilon) f(y) \over e(Y)} = {\exp(\epsilon) f(y) \over f(Y)} = \exp(\epsilon) q_F(y).\]

and

\[\mathbb P(E) = \int \mathbb P(E | \xi = y) p(y) dy = e(Y) \ge 1 - \delta,\]

and the same goes for \(\mathbb P(F)\).

What remains is to construct \(e(y)\) and \(f(y)\) satisfying the four
conditions.

Like in the proof of Claim 1, let \(S, T \subset Y\) be defined as

$$\begin{aligned}
S := \{y: p(y) > \exp(\epsilon) q(y)\},\\
T := \{y: q(y) > \exp(\epsilon) p(y)\}.
\end{aligned}$$

Let

$$\begin{aligned}
e(y) &:= \exp(\epsilon) q(y) 1_{y \in S} + p(y) 1_{y \notin S}\\
f(y) &:= \exp(\epsilon) p(y) 1_{y \in T} + q(y) 1_{y \notin T}. \qquad (6)
\end{aligned}$$

By checking them on the three disjoint subsets \(S\), \(T\), \((S \cup T)^c\),
it is not hard to verify that the \(e(y)\) and \(f(y)\) constructed this way
satisfy the first two conditions. They also satisfy the third condition:

$$\begin{aligned}
e(Y) &= 1 - (p(S) - \exp(\epsilon) q(S)) \ge 1 - \delta, \\
f(Y) &= 1 - (q(T) - \exp(\epsilon) p(T)) \ge 1 - \delta.
\end{aligned}$$

If \(e(Y) = f(Y)\) then we are done. Otherwise, without loss of
generality, assume \(e(Y) < f(Y)\), then all it remains to do is to reduce
the value of \(f(y)\) while preserving Condition 1, 2 and 3, until
\(f(Y) = e(Y)\).

As it turns out, this can be achieved by reducing \(f(y)\) on the set
\(\{y \in Y: q(y) > p(y)\}\). To see this, let us rename the \(f(y)\)
defined in (6) \(f_+(y)\), and construct \(f_-(y)\) by

\[f_-(y) := p(y) 1_{y \in T} + (q(y) \wedge p(y)) 1_{y \notin T}.\]

It is not hard to show that not only \(e(y)\) and \(f_-(y)\) also satisfy
conditions 1-3, but

\[e(y) \ge f_-(y), \forall y \in Y,\]

and thus \(e(Y) \ge f_-(Y)\). Therefore there exists an \(f\) that
interpolates between \(f_-\) and \(f_+\) with \(f(Y) = e(Y)\). \(\square\)

To prove the adaptive composition theorem for approximate differential
privacy, we need a similar claim (We use index shorthand
\(\xi_{< i} = \xi_{1 : i - 1}\) and similarly for other notations):

*Claim 5*. Let \(\xi_{1 : i}\) and \(\eta_{1 : i}\) be random variables. Let

$$\begin{aligned}
p_i(S | y_{1 : i - 1}) := \mathbb P(\xi_i \in S | \xi_{1 : i - 1} = y_{1 : i - 1})\\
q_i(S | y_{1 : i - 1}) := \mathbb P(\eta_i \in S | \eta_{1 : i - 1} = y_{1 : i - 1})
\end{aligned}$$

be the conditional laws of \(\xi_i | \xi_{< i}\) and \(\eta_i | \eta_{< i}\)
respectively. Then the following are equivalent:

1. For any \(y_{< i} \in Y^{i - 1}\), \(p_i(\cdot | y_{< i})\) and
   \(q_i(\cdot | y_{< i})\) are \((\epsilon, \delta)\)-ind

2. There exists events \(E_i, F_i \subset \Omega\) with
   \(\mathbb P(E_i | \xi_{< i} = y_{< i}) = \mathbb P(F_i | \eta_{< i} = y_{< i}) \ge 1 - \delta\)
   for any \(y_{< i}\), such that \(p_{i | E_i}(\cdot | y_{< i})\) and
   \(q_{i | E_i} (\cdot | y_{< i})\) are \(\epsilon\)-ind for any \(y_{< i}\),
   where $$\begin{aligned}
   p_{i | E_i}(S | y_{1 : i - 1}) := \mathbb P(\xi_i \in S | E_i, \xi_{1 : i - 1} = y_{1 : i - 1})\\
       q_{i | F_i}(S | y_{1 : i - 1}) := \mathbb P(\eta_i \in S | F_i, \eta_{1 : i - 1} = y_{1 : i - 1})
   \end{aligned}$$

   are \(p_i\) and \(q_i\) conditioned on \(E_i\) and \(F_i\) respectively.

*Proof*. Item 2 => Item 1: as in the Proof of Claim 4,

$$\begin{aligned}
p_i(S | y_{< i}) &= p_{i | E_i} (S | y_{< i}) \mathbb P(E_i | \xi_{< i} = y_{< i}) + p_{i | E_i^c}(S | y_{< i}) \mathbb P(E_i^c | \xi_{< i} = y_{< i}) \\
&\le p_{i | E_i} (S | y_{< i}) \mathbb P(E_i | \xi_{< i} = y_{< i}) + \delta \\
&= p_{i | E_i} (S | y_{< i}) \mathbb P(F_i | \xi_{< i} = y_{< i}) + \delta \\
&\le e^\epsilon q_{i | F_i} (S | y_{< i}) \mathbb P(F_i | \xi_{< i} = y_{< i}) + \delta \\
&= e^\epsilon q_i (S | y_{< i}) + \delta.
\end{aligned}$$

The direction from
\(q_i(S | y_{< i}) \le e^\epsilon p_i(S | y_{< i}) + \delta\) can be shown
in the same way.

Item 1 => Item 2: as in the Proof of Claim 4 we construct \(e(y_{1 : i})\)
and \(f(y_{1 : i})\) as "densities" of events \(E_i\) and \(F_i\).

Let

$$\begin{aligned}
e(y_{1 : i}) &:= e^\epsilon q_i(y_i | y_{< i}) 1_{y_i \in S_i(y_{< i})} + p_i(y_i | y_{< i}) 1_{y_i \notin S_i(y_{< i})}\\
f(y_{1 : i}) &:= e^\epsilon p_i(y_i | y_{< i}) 1_{y_i \in T_i(y_{< i})} + q_i(y_i | y_{< i}) 1_{y_i \notin T_i(y_{< i})}\\
\end{aligned}$$

where

$$\begin{aligned}
S_i(y_{< i}) = \{y_i \in Y: p_i(y_i | y_{< i}) > e^\epsilon q_i(y_i | y_{< i})\}\\
T_i(y_{< i}) = \{y_i \in Y: q_i(y_i | y_{< i}) > e^\epsilon p_i(y_i | y_{< i})\}.
\end{aligned}$$

Then \(E_i\) and \(F_i\) are defined as

$$\begin{aligned}
\mathbb P(E_i | \xi_{\le i} = y_{\le i}) &= {e(y_{\le i}) \over p_i(y_{\le i})},\\
\mathbb P(F_i | \xi_{\le i} = y_{\le i}) &= {f(y_{\le i}) \over q_i(y_{\le i})}.
\end{aligned}$$

The rest of the proof is almost the same as the proof of Claim 4.
\(\square\)

*** Back to approximate differential privacy
    :PROPERTIES:
    :CUSTOM_ID: back-to-approximate-differential-privacy
    :ID:       706c037d-ea44-4ade-8007-7f1f41d394e8
    :END:
By Claim 0 and 1 we have

*Claim 6*. If for all \(x, x' \in X\) with distance \(1\)

\[\mathbb P(L(M(x) || M(x')) \le \epsilon) \ge 1 - \delta,\]

then \(M\) is \((\epsilon, \delta)\)-dp.

Note that in the literature the divergence variable \(L(M(x) || M(x'))\)
is also called the /privacy loss/.

By Claim 0 and Claim 4 we have

*Claim 7*. \(M\) is \((\epsilon, \delta)\)-dp if and only if for every
\(x, x' \in X\) with distance \(1\), there exist events
\(E, F \subset \Omega\) with \(\mathbb P(E) = \mathbb P(F) \ge 1 - \delta\),
\(M(x) | E\) and \(M(x') | F\) are \(\epsilon\)-ind.

We can further simplify the privacy loss \(L(M(x) || M(x'))\), by
observing the translational and scaling invariance of \(L(\cdot||\cdot)\):

$$\begin{aligned}
L(\xi || \eta) &\overset{d}{=} L(\alpha \xi + \beta || \alpha \eta + \beta), \qquad \alpha \neq 0. \qquad (6.1)
\end{aligned}$$

With this and the definition

\[M(x) = f(x) + \zeta\]

for some random variable \(\zeta\), we have

\[L(M(x) || M(x')) \overset{d}{=} L(\zeta || \zeta + f(x') - f(x)).\]

Without loss of generality, we can consider \(f\) with sensitivity \(1\),
for

\[L(f(x) + S_f \zeta || f(x') + S_f \zeta) \overset{d}{=} L(S_f^{-1} f(x) + \zeta || S_f^{-1} f(x') + \zeta)\]

so for any noise \(\zeta\) that achieves \((\epsilon, \delta)\)-dp for a
function with sensitivity \(1\), we have the same privacy guarantee by for
an arbitrary function with sensitivity \(S_f\) by adding a noise
\(S_f \zeta\).

With Claim 6 we can show that the Gaussian mechanism is approximately
differentially private. But first we need to define it.

*Definition (Gaussian mechanism)*. Given a query \(f: X \to Y\), the
/Gaussian mechanism/ \(M\) adds a Gaussian noise to the query:

\[M(x) = f(x) + N(0, \sigma^2 I).\]

Some tail bounds for the Gaussian distribution will be useful.

*Claim 8 (Gaussian tail bounds)*. Let \(\xi \sim N(0, 1)\) be a standard
normal distribution. Then for \(t > 0\)

\[\mathbb P(\xi > t) < {1 \over \sqrt{2 \pi} t} e^{- {t^2 \over 2}}, \qquad (6.3)\]

and

\[\mathbb P(\xi > t) < e^{- {t^2 \over 2}}. \qquad (6.5)\]

*Proof*. Both bounds are well known. The first can be proved using

\[\int_t^\infty e^{- {y^2 \over 2}} dy < \int_t^\infty {y \over t} e^{- {y^2 \over 2}} dy.\]

The second is shown using
[[https://en.wikipedia.org/wiki/Chernoff_bound][Chernoff bound]]. For
any random variable \(\xi\),

\[\mathbb P(\xi > t) < {\mathbb E \exp(\lambda \xi) \over \exp(\lambda t)} = \exp(\kappa_\xi(\lambda) - \lambda t), \qquad (6.7)\]

where \(\kappa_\xi(\lambda) = \log \mathbb E \exp(\lambda \xi)\) is the
cumulant of \(\xi\). Since (6.7) holds for any \(\lambda\), we can get the
best bound by minimising \(\kappa_\xi(\lambda) - \lambda t\) (a.k.a. the
[[https://en.wikipedia.org/wiki/Legendre_transformation][Legendre
transformation]]). When \(\xi\) is standard normal, we get (6.5).
\(\square\)

*Remark*. We will use the Chernoff bound extensively in the second part
of this post when considering Rényi differential privacy.

*Claim 9*. The Gaussian mechanism on a query \(f\) is
\((\epsilon, \delta)\)-dp, where

\[\delta = \exp(- (\epsilon \sigma / S_f - (2 \sigma / S_f)^{-1})^2 / 2). \qquad (6.8)\]

Conversely, to achieve give \((\epsilon, \delta)\)-dp, we may set

\[\sigma > \left(\epsilon^{-1} \sqrt{2 \log \delta^{-1}} + (2 \epsilon)^{- {1 \over 2}}\right) S_f \qquad (6.81)\]

or

\[\sigma > (\epsilon^{-1} (1 \vee \sqrt{(\log (2 \pi)^{-1} \delta^{-2})_+}) + (2 \epsilon)^{- {1 \over 2}}) S_f \qquad (6.82)\]

or

\[\sigma > \epsilon^{-1} \sqrt{\log e^\epsilon \delta^{-2}} S_f \qquad (6.83)\]

or

\[\sigma > \epsilon^{-1} (\sqrt{1 + \epsilon} \vee \sqrt{(\log e^\epsilon (2 \pi)^{-1} \delta^{-2})_+}) S_f. \qquad (6.84)\]

*Proof*. As discussed before we only need to consider the case where
\(S_f = 1\). Fix arbitrary \(x, x' \in X\) with \(d(x, x') = 1\). Let
\(\zeta = (\zeta_1, ..., \zeta_d) \sim N(0, I_d)\).

By Claim 6 it suffices to bound

\[\mathbb P(L(M(x) || M(x')) > \epsilon)\]

We have by the linear invariance of \(L\),

\[L(M(x) || M(x')) = L(f(x) + \sigma \zeta || f(x') + \sigma \zeta) \overset{d}{=} L(\zeta|| \zeta + \Delta / \sigma),\]

where \(\Delta := f(x') - f(x)\).

Plugging in the Gaussian density, we have

\[L(M(x) || M(x')) \overset{d}{=} \sum_i {\Delta_i \over \sigma} \zeta_i + \sum_i {\Delta_i^2 \over 2 \sigma^2} \overset{d}{=} {\|\Delta\|_2 \over \sigma} \xi + {\|\Delta\|_2^2 \over 2 \sigma^2}.\]

where \(\xi \sim N(0, 1)\).

Hence

\[\mathbb P(L(M(x) || M(x')) > \epsilon) = \mathbb P(\zeta > {\sigma \over \|\Delta\|_2} \epsilon - {\|\Delta\|_2 \over 2 \sigma}).\]

Since \(\|\Delta\|_2 \le S_f = 1\), we have

\[\mathbb P(L(M(x) || M(x')) > \epsilon) \le \mathbb P(\xi > \sigma \epsilon - (2 \sigma)^{-1}).\]

Thus the problem is reduced to the tail bound of a standard normal
distribution, so we can use Claim 8. Note that we implicitly require
\(\sigma > (2 \epsilon)^{- 1 / 2}\) here so that
\(\sigma \epsilon - (2 \sigma)^{-1} > 0\) and we can use the tail bounds.

Using (6.3) we have

\[\mathbb P(L(M(x) || M(x')) > \epsilon) < \exp(- (\epsilon \sigma - (2 \sigma)^{-1})^2 / 2).\]

This gives us (6.8).

To bound the right hand by \(\delta\), we require

\[\epsilon \sigma - {1 \over 2 \sigma} > \sqrt{2 \log \delta^{-1}}. \qquad (6.91)\]

Solving this inequality we have

\[\sigma > {\sqrt{2 \log \delta^{-1}} + \sqrt{2 \log \delta^{-1} + 2 \epsilon} \over 2 \epsilon}.\]

Using
\(\sqrt{2 \log \delta^{-1} + 2 \epsilon} \le \sqrt{2 \log \delta^{-1}} + \sqrt{2 \epsilon}\),
we can achieve the above inequality by having

\[\sigma > \epsilon^{-1} \sqrt{2 \log \delta^{-1}} + (2 \epsilon)^{-{1 \over 2}}.\]

This gives us (6.81).

Alternatively, we can use the concavity of \(\sqrt{\cdot}\):

\[(2 \epsilon)^{-1} (\sqrt{2 \log \delta^{-1}} + \sqrt{2 \log \delta^{-1} + 2 \epsilon}) \le \epsilon^{-1} \sqrt{\log e^\epsilon \delta^{-2}},\]

which gives us (6.83)

Back to (6.9), if we use (6.5) instead, we need

\[\log t + {t^2 \over 2} > \log {(2 \pi)^{- 1 / 2} \delta^{-1}}\]

where \(t = \epsilon \sigma - (2 \sigma)^{-1}\). This can be satisfied if

$$\begin{aligned}
t &> 1 \qquad (6.93)\\
t &> \sqrt{\log (2 \pi)^{-1} \delta^{-2}}. \qquad (6.95)
\end{aligned}$$

We can solve both inequalities as before and obtain

\[\sigma > \epsilon^{-1} (1 \vee \sqrt{(\log (2 \pi)^{-1} \delta^{-2})_+}) + (2 \epsilon)^{- {1 \over 2}},\]

or

\[\sigma > \epsilon^{-1}(\sqrt{1 + \epsilon} \vee \sqrt{(\log e^\epsilon (2 \pi)^{-1} \delta^{-2})_+}).\]

This gives us (6.82)(6.84). \(\square\)

When \(\epsilon \le \alpha\) is bounded, by (6.83) (6.84) we can require
either

\[\sigma > \epsilon^{-1} (\sqrt{\log e^\alpha \delta^{-2}}) S_f\]

or

\[\sigma > \epsilon^{-1} (\sqrt{1 + \alpha} \vee \sqrt{(\log (2 \pi)^{-1} e^\alpha \delta^{-2})_+}) S_f.\]

The second bound is similar to and slightly better than the one in
Theorem A.1 of Dwork-Roth 2013, where \(\alpha = 1\):

\[\sigma > \epsilon^{-1} \left({3 \over 2} \vee \sqrt{(2 \log {5 \over 4} \delta^{-1})_+}\right) S_f.\]

Note that the lower bound of \({3 \over 2}\) is implicitly required in the
proof of Theorem A.1.

** Composition theorems
   :PROPERTIES:
   :CUSTOM_ID: composition-theorems
   :ID:       b672a060-d886-4f07-92d2-1d92f5f4c0c8
   :END:
So far we have seen how a mechanism made of a single query plus a noise
can be proved to be differentially private. But we need to understand
the privacy when composing several mechanisms, combinatorially or
sequentially. Let us first define the combinatorial case:

*Definition (Independent composition)*. Let \(M_1, ..., M_k\) be \(k\)
mechanisms with independent noises. The mechanism \(M = (M_1, ..., M_k)\)
is called the /independent composition/ of \(M_{1 : k}\).

To define the adaptive composition, let us motivate it with an example
of gradient descent. Consider the loss function \(\ell(x; \theta)\) of a
neural network, where \(\theta\) is the parameter and \(x\) the input,
gradient descent updates its parameter \(\theta\) at each time \(t\):

\[\theta_{t} = \theta_{t - 1} - \alpha m^{-1} \sum_{i = 1 : m} \nabla_\theta \ell(x_i; \theta) |_{\theta = \theta_{t - 1}}.\]

We may add privacy by adding noise \(\zeta_t\) at each step:

\[\theta_{t} = \theta_{t - 1} - \alpha m^{-1} \sum_{i = 1 : m} \nabla_\theta \ell(x_i; \theta) |_{\theta = \theta_{t - 1}} + \zeta_t. \qquad (6.97)\]

Viewed as a sequence of mechanism, we have that at each time \(t\), the
mechanism \(M_t\) takes input \(x\), and outputs \(\theta_t\). But \(M_t\) also
depends on the output of the previous mechanism \(M_{t - 1}\). To this
end, we define the adaptive composition.

*Definition (Adaptive composition)*. Let
\(({M_i(y_{1 : i - 1})})_{i = 1 : k}\) be \(k\) mechanisms with independent
noises, where \(M_1\) has no parameter, \(M_2\) has one parameter in \(Y\),
\(M_3\) has two parameters in \(Y\) and so on. For \(x \in X\), define \(\xi_i\)
recursively by

$$\begin{aligned}
\xi_1 &:= M_1(x)\\
\xi_i &:= M_i(\xi_1, \xi_2, ..., \xi_{i - 1}) (x).
\end{aligned}$$

The /adaptive composition/ of \(M_{1 : k}\) is defined by
\(M(x) := (\xi_1, \xi_2, ..., \xi_k)\).

The definition of adaptive composition may look a bit complicated, but
the point is to describe \(k\) mechanisms such that for each \(i\), the
output of the first, second, ..., \(i - 1\)th mechanisms determine the
\(i\)th mechanism, like in the case of gradient descent.

It is not hard to write down the differentially private gradient descent
as a sequential composition:

\[M_t(\theta_{1 : t - 1})(x) = \theta_{t - 1} - \alpha m^{-1} \sum_{i = 1 : m} \nabla_\theta \ell(x_i; \theta) |_{\theta = \theta_{t - 1}} + \zeta_t.\]

In Dwork-Rothblum-Vadhan 2010 (see also Dwork-Roth 2013) the adaptive
composition is defined in a more general way, but the definition is
based on the same principle, and proofs in this post on adaptive
compositions carry over.

It is not hard to see that the adaptive composition degenerates to
independent composition when each \(M_i(y_{1 : i})\) evaluates to the same
mechanism regardless of \(y_{1 : i}\), in which case the \(\xi_i\)s are
independent.

In the following when discussing adaptive compositions we sometimes omit
the parameters for convenience without risk of ambiguity, and write
\(M_i(y_{1 : i})\) as \(M_i\), but keep in mind of the dependence on the
parameters.

It is time to state and prove the composition theorems. In this section
we consider \(2 \times 2 \times 2 = 8\) cases, i.e. situations of three
dimensions, where there are two choices in each dimension:

1. Composition of \(\epsilon\)-dp or more generally
   \((\epsilon, \delta)\)-dp mechanisms
2. Composition of independent or more generally adaptive mechanisms
3. Basic or advanced compositions

Note that in the first two dimensions the second choice is more general
than the first.

The proofs of these composition theorems will be laid out as follows:

1. Claim 10 - Basic composition theorem for \((\epsilon, \delta)\)-dp with
   adaptive mechanisms: by a direct proof with an induction argument
2. Claim 14 - Advanced composition theorem for \(\epsilon\)-dp with
   independent mechanisms: by factorising privacy loss and using
   [[https://en.wikipedia.org/wiki/Hoeffding%27s_inequality][Hoeffding's
   Inequality]]
3. Claim 16 - Advanced composition theorem for \(\epsilon\)-dp with
   adaptive mechanisms: by factorising privacy loss and using
   [[https://en.wikipedia.org/wiki/Azuma%27s_inequality][Azuma's
   Inequality]]
4. Claims 17 and 18 - Advanced composition theorem for
   \((\epsilon, \delta)\)-dp with independent / adaptive mechanisms: by
   using characterisations of \((\epsilon, \delta)\)-dp in Claims 4 and 5
   as an approximation of \(\epsilon\)-dp and then using Proofs in Item 2
   or 3.

*Claim 10 (Basic composition theorem).* Let \(M_{1 : k}\) be \(k\)
mechanisms with independent noises such that for each \(i\) and
\(y_{1 : i - 1}\), \(M_i(y_{1 : i - 1})\) is \((\epsilon_i, \delta_i)\)-dp.
Then the adpative composition of \(M_{1 : k}\) is
\((\sum_i \epsilon_i, \sum_i \delta_i)\)-dp.

*Proof (Dwork-Lei 2009, see also Dwork-Roth 2013 Appendix B.1)*. Let \(x\)
and \(x'\) be neighbouring points in \(X\). Let \(M\) be the adaptive
composition of \(M_{1 : k}\). Define

\[\xi_{1 : k} := M(x), \qquad \eta_{1 : k} := M(x').\]

Let \(p^i\) and \(q^i\) be the laws of \((\xi_{1 : i})\) and \((\eta_{1 : i})\)
respectively.

Let \(S_1, ..., S_k \subset Y\) and \(T_i := \prod_{j = 1 : i} S_j\). We use
two tricks.

1. Since \(\xi_i | \xi_{< i} = y_{< i}\) and
   \(\eta_i | \eta_{< i} = y_{< i}\) are \((\epsilon_i, \delta_i)\)-ind, and
   a probability is no greater than \(1\), $$\begin{aligned}
   \mathbb P(\xi_i \in S_i | \xi_{< i} = y_{< i}) &\le (e^{\epsilon_i} \mathbb P(\eta_i \in S_i | \eta_{< i} = y_{< i}) + \delta_i) \wedge 1 \\
       &\le (e^{\epsilon_i} \mathbb P(\eta_i \in S_i | \eta_{< i} = y_{< i}) + \delta_i) \wedge (1 + \delta_i) \\
       &= (e^{\epsilon_i} \mathbb P(\eta_i \in S_i | \eta_{< i} = y_{< i})  \wedge 1) + \delta_i
   \end{aligned}$$

2. Given \(p\) and \(q\) that are \((\epsilon, \delta)\)-ind, define
   \[\mu(x) = (p(x) - e^\epsilon q(x))_+.\]

   We have \[\mu(S) \le \delta, \forall S\]

   In the following we define
   \(\mu^{i - 1} = (p^{i - 1} - e^\epsilon q^{i - 1})_+\) for the same
   purpose.

We use an inductive argument to prove the theorem:

$$\begin{aligned}
\mathbb P(\xi_{\le i} \in T_i) &= \int_{T_{i - 1}} \mathbb P(\xi_i \in S_i | \xi_{< i} = y_{< i}) p^{i - 1} (y_{< i}) dy_{< i} \\
&\le \int_{T_{i - 1}} (e^{\epsilon_i} \mathbb P(\eta_i \in S_i | \eta_{< i} = y_{< i}) \wedge 1) p^{i - 1}(y_{< i}) dy_{< i} + \delta_i\\
&\le \int_{T_{i - 1}} (e^{\epsilon_i} \mathbb P(\eta_i \in S_i | \eta_{< i} = y_{< i}) \wedge 1) (e^{\epsilon_1 + ... + \epsilon_{i - 1}} q^{i - 1}(y_{< i}) + \mu^{i - 1} (y_{< i})) dy_{< i} + \delta_i\\
&\le \int_{T_{i - 1}} e^{\epsilon_i} \mathbb P(\eta_i \in S_i | \eta_{< i} = y_{< i}) e^{\epsilon_1 + ... + \epsilon_{i - 1}} q^{i - 1}(y_{< i}) dy_{< i} + \mu_{i - 1}(T_{i - 1}) + \delta_i\\
&\le e^{\epsilon_1 + ... + \epsilon_i} \mathbb P(\eta_{\le i} \in T_i) + \delta_1 + ... + \delta_{i - 1} + \delta_i.\\
\end{aligned}$$

In the second line we use Trick 1; in the third line we use the
induction assumption; in the fourth line we multiply the first term in
the first braket with first term in the second braket, and the second
term (i.e. \(1\)) in the first braket with the second term in the second
braket (i.e. the \(\mu\) term); in the last line we use Trick 2.

The base case \(i = 1\) is true since \(M_1\) is
\((\epsilon_1, \delta_1)\)-dp. \(\square\)

To prove the advanced composition theorem, we start with some lemmas.

*Claim 11*. If \(p\) and \(q\) are \(\epsilon\)-ind, then

\[D(p || q) + D(q || p) \le \epsilon(e^\epsilon - 1).\]

*Proof*. Since \(p\) and \(q\) are \(\epsilon\)-ind, we have
\(|\log p(x) - \log q(x)| \le \epsilon\) for all \(x\). Let
\(S := \{x: p(x) > q(x)\}\). Then we have on

$$\begin{aligned}
D(p || q) + D(q || p) &= \int (p(x) - q(x)) (\log p(x) - \log q(x)) dx\\ 
&= \int_S (p(x) - q(x)) (\log p(x) - \log q(x)) dx + \int_{S^c} (q(x) - p(x)) (\log q(x) - \log p(x)) dx\\
&\le \epsilon(\int_S p(x) - q(x) dx + \int_{S^c} q(x) - p(x) dx)
\end{aligned}$$

Since on \(S\) we have \(q(x) \le p(x) \le e^\epsilon q(x)\), and on \(S^c\)
we have \(p(x) \le q(x) \le e^\epsilon p(x)\), we obtain

\[D(p || q) + D(q || p) \le \epsilon \int_S (1 - e^{-\epsilon}) p(x) dx + \epsilon \int_{S^c} (e^{\epsilon} - 1) p(x) dx \le \epsilon (e^{\epsilon} - 1),\]

where in the last step we use \(e^\epsilon - 1 \ge 1 - e^{- \epsilon}\)
and \(p(S) + p(S^c) = 1\). \(\square\)

*Claim 12*. If \(p\) and \(q\) are \(\epsilon\)-ind, then

\[D(p || q) \le a(\epsilon) \ge D(q || p),\]

where

\[a(\epsilon) = \epsilon (e^\epsilon - 1) 1_{\epsilon \le \log 2} + \epsilon 1_{\epsilon > \log 2} \le (\log 2)^{-1} \epsilon^2 1_{\epsilon \le \log 2} + \epsilon 1_{\epsilon > \log 2}. \qquad (6.98)\]

*Proof*. Since \(p\) and \(q\) are \(\epsilon\)-ind, we have

\[D(p || q) = \mathbb E_{\xi \sim p} \log {p(\xi) \over q(\xi)} \le \max_y {\log p(y) \over \log q(y)} \le \epsilon.\]

Comparing the quantity in Claim 11 (\(\epsilon(e^\epsilon - 1)\)) with the
quantity above (\(\epsilon\)), we arrive at the conclusion. \(\square\)

*Claim 13
([[https://en.wikipedia.org/wiki/Hoeffding%27s_inequality][Hoeffding's
Inequality]])*. Let \(L_i\) be independent random variables with
\(|L_i| \le b\), and let \(L = L_1 + ... + L_k\), then for \(t > 0\),

\[\mathbb P(L - \mathbb E L \ge t) \le \exp(- {t^2 \over 2 k b^2}).\]

*Claim 14 (Advanced Independent Composition Theorem)* (\(\delta = 0\)).
Fix \(0 < \beta < 1\). Let \(M_1, ..., M_k\) be \(\epsilon\)-dp, then the
independent composition \(M\) of \(M_{1 : k}\) is
\((k a(\epsilon) + \sqrt{2 k \log \beta^{-1}} \epsilon, \beta)\)-dp.

*Remark*. By (6.98) we know that
\(k a(\epsilon) + \sqrt{2 k \log \beta^{-1}} \epsilon = \sqrt{2 k \log \beta^{-1}} \epsilon + k O(\epsilon^2)\)
when \(\epsilon\) is sufficiently small, in which case the leading term is
of order \(O(\sqrt k \epsilon)\) and we save a \(\sqrt k\) in the
\(\epsilon\)-part compared to the Basic Composition Theorem (Claim 10).

*Remark*. In practice one can try different choices of \(\beta\) and
settle with the one that gives the best privacy guarantee. See the
discussions at the end of
[[/posts/2019-03-14-great-but-manageable-expectations.html][Part 2 of
this post]].

*Proof*. Let \(p_i\), \(q_i\), \(p\) and \(q\) be the laws of \(M_i(x)\),
\(M_i(x')\), \(M(x)\) and \(M(x')\) respectively.

\[\mathbb E L_i = D(p_i || q_i) \le a(\epsilon),\]

where \(L_i := L(p_i || q_i)\). Due to \(\epsilon\)-ind also have

\[|L_i| \le \epsilon.\]

Therefore, by Hoeffding's Inequality,

\[\mathbb P(L - k a(\epsilon) \ge t) \le \mathbb P(L - \mathbb E L \ge t) \le \exp(- t^2 / 2 k \epsilon^2),\]

where \(L := \sum_i L_i = L(p || q)\).

Plugging in \(t = \sqrt{2 k \epsilon^2 \log \beta^{-1}}\), we have

\[\mathbb P(L(p || q) \le k a(\epsilon) + \sqrt{2 k \epsilon^2 \log \beta^{-1}}) \ge 1 - \beta.\]

Similarly we also have

\[\mathbb P(L(q || p) \le k a(\epsilon) + \sqrt{2 k \epsilon^2 \log \beta^{-1}}) \ge 1 - \beta.\]

By Claim 1 we arrive at the conclusion. \(\square\)

*Claim 15 ([[https://en.wikipedia.org/wiki/Azuma%27s_inequality][Azuma's
Inequality]])*. Let \(X_{0 : k}\) be a
[[https://en.wikipedia.org/wiki/Martingale_(probability_theory)][supermartingale]].
If \(|X_i - X_{i - 1}| \le b\), then

\[\mathbb P(X_k - X_0 \ge t) \le \exp(- {t^2 \over 2 k b^2}).\]

Azuma's Inequality implies a slightly weaker version of Hoeffding's
Inequality. To see this, let \(L_{1 : k}\) be independent variables with
\(|L_i| \le b\). Let \(X_i = \sum_{j = 1 : i} L_j - \mathbb E L_j\). Then
\(X_{0 : k}\) is a martingale, and

\[| X_i - X_{i - 1} | = | L_i - \mathbb E L_i | \le 2 b,\]

since \(\|L_i\|_1 \le \|L_i\|_\infty\). Hence by Azuma's Inequality,

\[\mathbb P(L - \mathbb E L \ge t) \le \exp(- {t^2 \over 8 k b^2}).\]

Of course here we have made no assumption on \(\mathbb E L_i\). If instead
we have some bound for the expectation, say \(|\mathbb E L_i| \le a\),
then by the same derivation we have

\[\mathbb P(L - \mathbb E L \ge t) \le \exp(- {t^2 \over 2 k (a + b)^2}).\]

It is not hard to see what Azuma is to Hoeffding is like adaptive
composition to independent composition. Indeed, we can use Azuma's
Inequality to prove the Advanced Adaptive Composition Theorem for
\(\delta = 0\).

*Claim 16 (Advanced Adaptive Composition Theorem)* (\(\delta = 0\)). Let
\(\beta > 0\). Let \(M_{1 : k}\) be \(k\) mechanisms with independent noises
such that for each \(i\) and \(y_{1 : i}\), \(M_i(y_{1 : i})\) is
\((\epsilon, 0)\)-dp. Then the adpative composition of \(M_{1 : k}\) is
\((k a(\epsilon) + \sqrt{2 k \log \beta^{-1}} (\epsilon + a(\epsilon)), \beta)\)-dp.

*Proof*. As before, let \(\xi_{1 : k} \overset{d}{=} M(x)\) and
\(\eta_{1 : k} \overset{d}{=} M(x')\), where \(M\) is the adaptive
composition of \(M_{1 : k}\). Let \(p_i\) (resp. \(q_i\)) be the law of
\(\xi_i | \xi_{< i}\) (resp. \(\eta_i | \eta_{< i}\)). Let \(p^i\) (resp.
\(q^i\)) be the law of \(\xi_{\le i}\) (resp. \(\eta_{\le i}\)). We want to
construct supermartingale \(X\). To this end, let

\[X_i = \log {p^i(\xi_{\le i}) \over q^i(\xi_{\le i})} - i a(\epsilon) \]

We show that \((X_i)\) is a supermartingale:

$$\begin{aligned}
\mathbb E(X_i - X_{i - 1} | X_{i - 1}) &= \mathbb E \left(\log {p_i (\xi_i | \xi_{< i}) \over q_i (\xi_i | \xi_{< i})} - a(\epsilon) | \log {p^{i - 1} (\xi_{< i}) \over q^{i - 1} (\xi_{< i})}\right) \\
&= \mathbb E \left( \mathbb E \left(\log {p_i (\xi_i | \xi_{< i}) \over q_i (\xi_i | \xi_{< i})} | \xi_{< i}\right) | \log {p^{i - 1} (\xi_{< i}) \over q^{i - 1} (\xi_{< i})}\right) - a(\epsilon) \\
&= \mathbb E \left( D(p_i (\cdot | \xi_{< i}) || q_i (\cdot | \xi_{< i})) | \log {p^{i - 1} (\xi_{< i}) \over q^{i - 1} (\xi_{< i})}\right) - a(\epsilon) \\
&\le 0,
\end{aligned}$$

since by Claim 12
\(D(p_i(\cdot | y_{< i}) || q_i(\cdot | y_{< i})) \le a(\epsilon)\) for
all \(y_{< i}\).

Since

\[| X_i - X_{i - 1} | = | \log {p_i(\xi_i | \xi_{< i}) \over q_i(\xi_i | \xi_{< i})} - a(\epsilon) | \le \epsilon + a(\epsilon),\]

by Azuma's Inequality,

\[\mathbb P(\log {p^k(\xi_{1 : k}) \over q^k(\xi_{1 : k})} \ge k a(\epsilon) + t) \le \exp(- {t^2 \over 2 k (\epsilon + a(\epsilon))^2}). \qquad(6.99)\]

Let \(t = \sqrt{2 k \log \beta^{-1}} (\epsilon + a(\epsilon))\) we are
done. \(\square\)

*Claim 17 (Advanced Independent Composition Theorem)*. Fix
\(0 < \beta < 1\). Let \(M_1, ..., M_k\) be \((\epsilon, \delta)\)-dp, then
the independent composition \(M\) of \(M_{1 : k}\) is
\((k a(\epsilon) + \sqrt{2 k \log \beta^{-1}} \epsilon, k \delta + \beta)\)-dp.

*Proof*. By Claim 4, there exist events \(E_{1 : k}\) and \(F_{1 : k}\) such
that

1. The laws \(p_{i | E_i}\) and \(q_{i | F_i}\) are \(\epsilon\)-ind.
2. \(\mathbb P(E_i), \mathbb P(F_i) \ge 1 - \delta\).

Let \(E := \bigcap E_i\) and \(F := \bigcap F_i\), then they both have
probability at least \(1 - k \delta\), and \(p_{i | E}\) and \(q_{i | F}\) are
\(\epsilon\)-ind.

By Claim 14, \(p_{|E}\) and \(q_{|F}\) are
\((\epsilon' := k a(\epsilon) + \sqrt{2 k \epsilon^2 \log \beta^{-1}}, \beta)\)-ind.
Let us shrink the bigger event between \(E\) and \(F\) so that they have
equal probabilities. Then

$$\begin{aligned}
p (S) &\le p_{|E}(S) \mathbb P(E) + \mathbb P(E^c) \\
&\le (e^{\epsilon'} q_{|F}(S) + \beta) \mathbb P(F) + k \delta\\
&\le e^{\epsilon'} q(S) + \beta + k \delta.
\end{aligned}$$

\(\square\)

*Claim 18 (Advanced Adaptive Composition Theorem)*. Fix \(0 < \beta < 1\).
Let \(M_{1 : k}\) be \(k\) mechanisms with independent noises such that for
each \(i\) and \(y_{1 : i}\), \(M_i(y_{1 : i})\) is \((\epsilon, \delta)\)-dp.
Then the adpative composition of \(M_{1 : k}\) is
\((k a(\epsilon) + \sqrt{2 k \log \beta^{-1}} (\epsilon + a(\epsilon)), \beta + k \delta)\)-dp.

*Remark*. This theorem appeared in Dwork-Rothblum-Vadhan 2010, but I
could not find a proof there. A proof can be found in Dwork-Roth 2013
(See Theorem 3.20 there). Here I prove it in a similar way, except that
instead of the use of an intermediate random variable there, I use the
conditional probability results from Claim 5, the approach mentioned in
Vadhan 2017.

*Proof*. By Claim 5, there exist events \(E_{1 : k}\) and \(F_{1 : k}\) such
that

1. The laws \(p_{i | E_i}(\cdot | y_{< i})\) and
   \(q_{i | F_i}(\cdot | y_{< i})\) are \(\epsilon\)-ind for all \(y_{< i}\).
2. \(\mathbb P(E_i | y_{< i}), \mathbb P(F_i | y_{< i}) \ge 1 - \delta\)
   for all \(y_{< i}\).

Let \(E := \bigcap E_i\) and \(F := \bigcap F_i\), then they both have
probability at least \(1 - k \delta\), and \(p_{i | E}(\cdot | y_{< i}\) and
\(q_{i | F}(\cdot | y_{< i})\) are \(\epsilon\)-ind.

By Advanced Adaptive Composition Theorem (\(\delta = 0\)), \(p_{|E}\) and
\(q_{|F}\) are
\((\epsilon' := k a(\epsilon) + \sqrt{2 k \log \beta^{-1}} (\epsilon + a(\epsilon)), \beta)\)-ind.

The rest is the same as in the proof of Claim 17. \(\square\)

** Subsampling
   :PROPERTIES:
   :CUSTOM_ID: subsampling
   :ID:       eeda51d4-9370-49c6-9710-9c9ab88f91e2
   :END:
Stochastic gradient descent is like gradient descent, but with random
subsampling.

Recall we have been considering databases in the space \(Z^m\). Let
\(n < m\) be a positive integer,
\(\mathcal I := \{I \subset [m]: |I| = n\}\) be the set of subsets of
\([m]\) of size \(n\), and \(\gamma\) a random subset sampled uniformly from
\(\mathcal I\). Let \(r = {n \over m}\) which we call the subsampling rate.
Then we may add a subsampling module to the noisy gradient descent
algorithm (6.97) considered before

\[\theta_{t} = \theta_{t - 1} - \alpha n^{-1} \sum_{i \in \gamma} \nabla_\theta h_\theta(x_i) |_{\theta = \theta_{t - 1}} + \zeta_t. \qquad (7)\]

It turns out subsampling has an amplification effect on privacy.

*Claim 19 (Ullman 2017)*. Fix \(r \in [0, 1]\). Let \(n \le m\) be two
nonnegative integers with \(n = r m\). Let \(N\) be an
\((\epsilon, \delta)\)-dp mechanism on \(Z^n\). Define mechanism \(M\) on
\(Z^m\) by

\[M(x) = N(x_\gamma)\]

Then \(M\) is \((\log (1 + r(e^\epsilon - 1)), r \delta)\)-dp.

*Remark*. Some seem to cite
Kasiviswanathan-Lee-Nissim-Raskhodnikova-Smith 2005 for this result, but
it is not clear to me how it appears there.

*Proof*. Let \(x, x' \in Z^n\) such that they differ by one row
\(x_i \neq x_i'\). Naturally we would like to consider the cases where the
index \(i\) is picked and the ones where it is not separately. Let
\(\mathcal I_\in\) and \(\mathcal I_\notin\) be these two cases:

$$\begin{aligned}
\mathcal I_\in = \{J \subset \mathcal I: i \in J\}\\
\mathcal I_\notin = \{J \subset \mathcal I: i \notin J\}\\
\end{aligned}$$

We will use these notations later. Let \(A\) be the event
\(\{\gamma \ni i\}\).

Let \(p\) and \(q\) be the laws of \(M(x)\) and \(M(x')\) respectively. We
collect some useful facts about them. First due to \(N\) being
\((\epsilon, \delta)\)-dp,

\[p_{|A}(S) \le e^\epsilon q_{|A}(S) + \delta.\]

Also,

\[p_{|A}(S) \le e^\epsilon p_{|A^c}(S) + \delta.\]

To see this, note that being conditional laws, \(p_A\) and \(p_{A^c}\) are
averages of laws over \(\mathcal I_\in\) and \(\mathcal I_\notin\)
respectively:

$$\begin{aligned}
p_{|A}(S) = |\mathcal I_\in|^{-1} \sum_{I \in \mathcal I_\in} \mathbb P(N(x_I) \in S)\\
p_{|A^c}(S) = |\mathcal I_\notin|^{-1} \sum_{J \in \mathcal I_\notin} \mathbb P(N(x_J) \in S).
\end{aligned}$$

Now we want to pair the \(I\)'s in \(\mathcal I_\in\) and \(J\)'s in
\(\mathcal I_\notin\) so that they differ by one index only, which means
\(d(x_I, x_J) = 1\). Formally, this means we want to consider the set:

\[\mathcal D := \{(I, J) \in \mathcal I_\in \times \mathcal I_\notin: |I \cap J| = n - 1\}.\]

We may observe by trying out some simple cases that every
\(I \in \mathcal I_\in\) is paired with \(n\) elements in
\(\mathcal I_\notin\), and every \(J \in \mathcal I_\notin\) is paired with
\(m - n\) elements in \(\mathcal I_\in\). Therefore

\[p_{|A}(S) = |\mathcal D|^{-1} \sum_{(I, J) \in \mathcal D} \mathbb P(N(x_I \in S)) \le |\mathcal D|^{-1} \sum_{(I, J) \in \mathcal D} (e^\epsilon \mathbb P(N(x_J \in S)) + \delta) = e^\epsilon p_{|A^c} (S) + \delta.\]

Since each of the \(m\) indices is picked independently with probability
\(r\), we have

\[\mathbb P(A) = r.\]

Let \(t \in [0, 1]\) to be determined. We may write

$$\begin{aligned}
p(S) &= r p_{|A} (S) + (1 - r) p_{|A^c} (S)\\
&\le r(t e^\epsilon q_{|A}(S) + (1 - t) e^\epsilon q_{|A^c}(S) + \delta) + (1 - r) q_{|A^c} (S)\\
&= rte^\epsilon q_{|A}(S) + (r(1 - t) e^\epsilon + (1 - r)) q_{|A^c} (S) + r \delta\\
&= te^\epsilon r q_{|A}(S) + \left({r \over 1 - r}(1 - t) e^\epsilon + 1\right) (1 - r) q_{|A^c} (S) + r \delta \\
&\le \left(t e^\epsilon \wedge \left({r \over 1 - r} (1 - t) e^\epsilon + 1\right)\right) q(S) + r \delta. \qquad (7.5)
\end{aligned}$$

We can see from the last line that the best bound we can get is when

\[t e^\epsilon = {r \over 1 - r} (1 - t) e^\epsilon + 1.\]

Solving this equation we obtain

\[t = r + e^{- \epsilon} - r e^{- \epsilon}\]

and plugging this in (7.5) we have

\[p(S) \le (1 + r(e^\epsilon - 1)) q(S) + r \delta.\]

\(\square\)

Since \(\log (1 + x) < x\) for \(x > 0\), we can rewrite the conclusion of
the Claim to \((r(e^\epsilon - 1), r \delta)\)-dp. Further more, if
\(\epsilon < \alpha\) for some \(\alpha\), we can rewrite it as
\((r \alpha^{-1} (e^\alpha - 1) \epsilon, r \delta)\)-dp or
\((O(r \epsilon), r \delta)\)-dp.

Let \(\epsilon < 1\). We see that if the mechanism \(N\) is
\((\epsilon, \delta)\)-dp on \(Z^n\), then \(M\) is
\((2 r \epsilon, r \delta)\)-dp, and if we run it over \(k / r\)
minibatches, by Advanced Adaptive Composition theorem, we have
\((\sqrt{2 k r \log \beta^{-1}} \epsilon + 2 k r \epsilon^2, k \delta + \beta)\)-dp.

This is better than the privacy guarantee without subsampling, where we
run over \(k\) iterations and obtain
\((\sqrt{2 k \log \beta^{-1}} \epsilon + 2 k \epsilon^2, k \delta + \beta)\)-dp.
So with subsampling we gain an extra \(\sqrt r\) in the \(\epsilon\)-part of
the privacy guarantee. But, smaller subsampling rate means smaller
minibatch size, which would result in bigger variance, so there is a
trade-off here.

Finally we define the differentially private stochastic gradient descent
(DP-SGD) with the Gaussian mechanism
(Abadi-Chu-Goodfellow-McMahan-Mironov-Talwar-Zhang 2016), which is (7)
with the noise specialised to Gaussian and an added clipping operation
to bound to sensitivity of the query to a chosen \(C\):

\[\theta_{t} = \theta_{t - 1} - \alpha \left(n^{-1} \sum_{i \in \gamma} \nabla_\theta \ell(x_i; \theta) |_{\theta = \theta_{t - 1}}\right)_{\text{Clipped at }C / 2} + N(0, \sigma^2 C^2 I),\]

where

\[y_{\text{Clipped at } \alpha} := y / (1 \vee {\|y\|_2 \over \alpha})\]

is \(y\) clipped to have norm at most \(\alpha\).

Note that the clipping in DP-SGD is much stronger than making the query
have sensitivity \(C\). It makes the difference between the query results
of two /arbitrary/ inputs bounded by \(C\), rather than /neighbouring/
inputs.

In [[/posts/2019-03-14-great-but-manageable-expectations.html][Part 2 of
this post]] we will use the tools developed above to discuss the privacy
guarantee for DP-SGD, among other things.

** References
   :PROPERTIES:
   :CUSTOM_ID: references
   :ID:       65686625-6bd1-4e42-b00d-5f1744945884
   :END:

- Abadi, Martín, Andy Chu, Ian Goodfellow, H. Brendan McMahan, Ilya
  Mironov, Kunal Talwar, and Li Zhang. "Deep Learning with Differential
  Privacy." Proceedings of the 2016 ACM SIGSAC Conference on Computer
  and Communications Security - CCS'16, 2016, 308--18.
  [[https://doi.org/10.1145/2976749.2978318]].
- Dwork, Cynthia, and Aaron Roth. "The Algorithmic Foundations of
  Differential Privacy." Foundations and Trends® in Theoretical Computer
  Science 9, no. 3--4 (2013): 211--407.
  [[https://doi.org/10.1561/0400000042]].
- Dwork, Cynthia, Guy N. Rothblum, and Salil Vadhan. "Boosting and
  Differential Privacy." In 2010 IEEE 51st Annual Symposium on
  Foundations of Computer Science, 51--60. Las Vegas, NV, USA:
  IEEE, 2010. [[https://doi.org/10.1109/FOCS.2010.12]].
- Shiva Prasad Kasiviswanathan, Homin K. Lee, Kobbi Nissim, Sofya
  Raskhodnikova, and Adam Smith. "What Can We Learn Privately?" In 46th
  Annual IEEE Symposium on Foundations of Computer Science (FOCS'05).
  Pittsburgh, PA, USA: IEEE, 2005.
  [[https://doi.org/10.1109/SFCS.2005.1]].
- Murtagh, Jack, and Salil Vadhan. "The Complexity of Computing the
  Optimal Composition of Differential Privacy." In Theory of
  Cryptography, edited by Eyal Kushilevitz and Tal Malkin, 9562:157--75.
  Berlin, Heidelberg: Springer Berlin Heidelberg, 2016.
  [[https://doi.org/10.1007/978-3-662-49096-9_7]].
- Ullman, Jonathan. "Solution to CS7880 Homework 1.", 2017.
  [[http://www.ccs.neu.edu/home/jullman/cs7880s17/HW1sol.pdf]]
- Vadhan, Salil. "The Complexity of Differential Privacy." In Tutorials
  on the Foundations of Cryptography, edited by Yehuda Lindell,
  347--450. Cham: Springer International Publishing, 2017.
  [[https://doi.org/10.1007/978-3-319-57048-8_7]].

[fn:1] For those who have read about differential privacy and never
       heard of the term "divergence variable", it is closely related to
       the notion of "privacy loss", see the paragraph under Claim 6 in
       [[#back-to-approximate-differential-privacy][Back to approximate
       differential privacy]]. I defined the term this way so that we
       can focus on the more general stuff: compared to the privacy loss
       \(L(M(x) || M(x'))\), the term \(L(p || q)\) removes the "distracting
       information" that \(p\) and \(q\) are related to databases, queries,
       mechanisms etc., but merely probability laws. By removing the
       distraction, we simplify the analysis. And once we are done with
       the analysis of \(L(p || q)\), we can apply the results obtained in
       the general setting to the special case where \(p\) is the law of
       \(M(x)\) and \(q\) is the law of \(M(x')\).